Đến nội dung

dogsteven nội dung

Có 1000 mục bởi dogsteven (Tìm giới hạn từ 29-04-2020)



Sắp theo                Sắp xếp  

#546893 $S^2 \le k(a^4+b^4+c^4)$

Đã gửi bởi dogsteven on 13-03-2015 - 16:30 trong Bất đẳng thức và cực trị

Ta có $16k\geqslant \dfrac{(a+b+c)(a+b-c)(b+c-a)(c+a-b)}{a^4+b^4+c^4}=f$

Vậy ta có $16k_{min}=f_{max}$. Dễ thấy $f\leqslant \dfrac{abc(a+b+c)}{a^4+b^4+c^4}\leqslant 1$ nên $k_{min}=\dfrac{1}{16}$




#546880 Chứng minh $\sum \frac{h_{a}}{h_...

Đã gửi bởi dogsteven on 13-03-2015 - 14:52 trong Hình học

Đặt $(h_a, h_b, h_c)=(x,y,z)$

$\hat{A}\geqslant \hat{B}\geqslant \hat{C}\Leftrightarrow a\geqslant b\geqslant c\Leftrightarrow x\leqslant y\leqslant z$

Bất đẳng thức tương đương với $(x-y)(y-z)(z-x)\geqslant 0$ luôn đúng.




#546879 Chứng minh $\sqrt{x}+\sqrt{y}+\sqrt...

Đã gửi bởi dogsteven on 13-03-2015 - 14:48 trong Hình học

Áp dụng bất đẳng thức Cauchy-Schwarz

$\sum \sqrt{bc}. \sqrt{\dfrac{ax}{2}} \leqslant \sqrt{(ab+bc+ca).S_{ABC}}\leqslant \sqrt{\dfrac{abc(a^2+b^2+c^2)}{4R}}$

Do đó ta có điều phải chứng minh.




#546876 Chứng minh $a+b+c\geq 2.\sqrt[4]{27}.\sqrt...

Đã gửi bởi dogsteven on 13-03-2015 - 14:41 trong Hình học

Mũ bốn cả hai vế, ta được bất đẳng thức tương đương $(a+b+c)^3\geqslant 27(b+c-a)(c+a-b)(a+b-c)$

Áp dụng bất đẳng thức AM-GM $(a+b+c)^3\geqslant 27abc$ nên ta cần chứng minh $abc\geqslant \prod (b+c-a)$

Bất đẳng thức này hiển nhiên đúng.




#546875 Chứng minh $\frac{ab+bc+ca}{2}\geq \s...

Đã gửi bởi dogsteven on 13-03-2015 - 14:40 trong Bất đẳng thức và cực trị

$LHS=\sum \dfrac{ab\sqrt{ab}}{2\sqrt{ab}}\geqslant \sum \dfrac{ab\sqrt{ab}}{a+b}$

Vậy cho $a,b,c$ là độ dài ba cạnh tam giác để làm gì.




#546874 Chứng minh $\frac{r}{R}\leq \frac...

Đã gửi bởi dogsteven on 13-03-2015 - 14:37 trong Hình học

Giả sử $c=\text{min}\{a,b,c\}$. Áp dụng bất đẳng thức Cauchy-Schwarz: $(a+b+c)^2(b+c-a)(c+a-b)(a+b-c)\leqslant 3(a^2+b^2+c^2)(b+c-a)(c+a-b)(a+b-c)$

Ta cần chứng minh $(a^2+b^2+c^2)(b+c-a)(c+a-b)(a+b-c)\leqslant abc(ab+bc+ca)$

$\Leftrightarrow (a^2+b^2+c^2)(a+b-c)(a-b)^2+c\left[(a-b)^2+c^2+ab\right](a-c)(b-c)\geqslant 0$ luôn đúng.




#546767 Chứng minh $\frac{r}{R}\leq \frac...

Đã gửi bởi dogsteven on 12-03-2015 - 20:14 trong Hình học

Bất đẳng thức tương đương với $(a+b+c)^2(b+c-a)(c+a-b)(a+b-c)\leqslant 3abc(ab+bc+ca)$

Bất đẳng thức này chứng minh đơn giản bằng $S-S$




#546761 CM: $x-y$ là số chính phương

Đã gửi bởi dogsteven on 12-03-2015 - 19:48 trong Số học

$3x^2-3y^3+x-y=y^2\Leftrightarrow (x-y)(3x+3y-1)=y^2$

Nếu $p$ là một số nguyên tố bất kỳ sao cho $p|x-y$ thì $p|y\Rightarrow p|x+y\Rightarrow p\nmid 3(x+y)+1$

Do đó $(x-y,3x+3y+1)=1$




#546309 $\sum \sqrt{\frac{a^{3}}{a^...

Đã gửi bởi dogsteven on 26-02-2015 - 15:31 trong Bất đẳng thức - Cực trị

$\sum \sqrt{\dfrac{a^3}{a^2+3b^2}}=\sum \dfrac{2a^2}{\sqrt{4a(a^2+3b^2)}} \geqslant \sum \dfrac{4a^2}{4a+a^2+3b^2}=12\sum \dfrac{a^2}{4a(a+b+c)+3(a^2+3b^2)}$

$\sum \dfrac{a^2}{4a(a+b+c)+3(a^2+3b^2)}\geqslant \dfrac{\sum (c+2a)^2\left[4a(a+b+c)+3(a^2+3b^2)\right]}{ (2a^2+2b^2+2c^2+ab+bc+ca)^2}$

Đến đây thì BW là được.




#546106 $61[(a+b)^6+(b+c)^6+(c+a)^6]\ge 16(a^6+b^6+c^6)$

Đã gửi bởi dogsteven on 25-02-2015 - 17:59 trong Bất đẳng thức - Cực trị

Cái pp đặt đó đó là sao ? :(?? 

Nếu như ta chuyển bất đẳng thức về một hàm đồng biến theo $r$ thì ta chỉ cần chứng minh bất đẳng thức khi hai biến bằng nhau. (Định lý ABC chứng minh bằng đại lượng $(a-b)^2(b-c)^2(c-a)^2$ hoặc bằng khảo sát hàm số) Định lý này dùng ở các bài toán mạnh.

Ở đây, họ chuyển bất đẳng thức về dạng hàm số bậc nhất theo $r$, hay theo $w^3$. Khi đó thì ta chỉ cần chứng minh bất đẳng thức khi $b=c$ và chuẩn hóa $b=c=1$

Lưu ý ở đây $3v^2=ab+bc+ca$ và nếu $ab+bc+ca<0$ thì $v$ là số phức. Chính là thiếu sót của cách đặt trên.




#546028 $\frac{a+b}{\sqrt{a^2+b}}+\...

Đã gửi bởi dogsteven on 25-02-2015 - 12:08 trong Bất đẳng thức và cực trị

Bài 3. Trước tiên ta chứng minh $a+b+c+\frac{5}{3}abc\geq 2$

Giả sử $a\geq b\geq c\geq 0\Rightarrow 0\leq bc\leq \frac{1}{3}$

Thay $a=\frac{1-bc}{b+c}$ thì ta cần chứng minh $\frac{1-bc}{b+c}+b+c+\frac{5}{3}.\frac{1-bc}{b+c}.bc\geq 2$

                          $\Leftrightarrow 3(1-bc)+3(b+c)^2+5(1-bc)bc-6(b+c)\geq 0$

                          $\Leftrightarrow 3(b+c-1)^2+bc(2-5bc)\geq 0$ (đúng)

Quay lại bài toán:

Ta cần c/m: $\frac{1}{a+b}+\frac{1}{b+c}+\frac{1}{c+a}\geq \frac{5}{2}$

                   $\Leftrightarrow 2(a+b)(b+c)+2(a+c)(b+c)+2(a+b)(a+c)\geq 5(a+b)(b+c)(c+a)$

                   $\Leftrightarrow 2(a^2+b^2+c^2+2ab+2bc+2ca)+2(ab+bc+ca)\geq 5(a+b+c)(ab+bc+ca)-5abc$

                   $\Leftrightarrow 2(a+b+c)^2+2+5abc\geq 5(a+b+c)$

Mà $a+b+c+\frac{5}{3}abc\geq 2\Rightarrow 5abc\geq 6-3(a+b+c)$

Nên ta chỉ cần c/m: $2(a+b+c)^2+2+6-3(a+b+c)\geq 5(a+b+c)$

                               $\Leftrightarrow 2(a+b+c)^2-8(a+b+c)+8\geq 0\Leftrightarrow 2(a+b+c-2)^2\geq 0$

Vậy ta có đpcm

Dấu "=" xảy ra khi $(a,b,c)=(1,1,0)$ và các hoán vị

Bất đẳng thức tương đương với $2(a+b+c)^2+2+5abc\geqslant 5(a+b+c)$

Nếu $a+b+c\geqslant 2$ thì bất đẳng thức hiển nhiên đúng.

Nếu $\sqrt{3}\leqslant a+b+c\leqslant 2$ thì theo bất đẳng thức Schur $5abc\geqslant \dfrac{5(a+b+c)[4-(a+b+c)^2]}{9}$ nên ta cần chứng minh:

$$-5(a+b+c)^3+18(a+b+c)^2-20(a+b+c)+18\geqslant 0\Leftrightarrow (2-a-b-c)\left[5(a+b+c)^2-8(a+b+c)+9\right]\geqslant 0$$




#545963 $\frac{a+b}{\sqrt{a^2+b}}+\...

Đã gửi bởi dogsteven on 24-02-2015 - 21:12 trong Bất đẳng thức và cực trị

Bài 1. Dễ.

Bài 2. Đặt $f(x)=x^3+\dfrac{1}{x^3}-17x$. giả sử $a=\text{max}\{a,b,c\}$ thì chứng minh $f(a)+f(b)+f(c)\geqslant f(a)+2f(\sqrt{bc}) \geqslant 0$

Còn một lời giải bằng chia để trị mà lười ghi.

Bài 3. Dồn biến hoặc pqr.

Bài 4. Yếu tố.




#545948 $\frac{a+b+c}{3}\geq \sqrt[11]{...

Đã gửi bởi dogsteven on 24-02-2015 - 20:35 trong Bất đẳng thức - Cực trị

Bài này uvw.




#545932 $\sum \frac{1}{11+a^2} \le \frac...

Đã gửi bởi dogsteven on 24-02-2015 - 19:42 trong Bất đẳng thức - Cực trị

Đúng mà bạn -_-

$a=1.97749, b=1.12331, c=0.468045\rightarrow d=0.431155$

$\frac{a+1}{a^2+11}-\frac{b+1}{b^2+11}=0.02652700087 $

$\frac{b+1}{b^2+11}-\frac{c+1}{c^2+11}=0.04231157535 $

$\frac{c+1}{c^2+11}-\frac{d+1}{d^2+11}=0.0022909863053$

Á chết. Tính nhầm $d=4-a+b+c$




#545921 $\sum \frac{1}{11+a^2} \le \frac...

Đã gửi bởi dogsteven on 24-02-2015 - 18:52 trong Bất đẳng thức - Cực trị

Ta có $\sum \frac{1}{11+a^2} \le \frac{1}{3}\Leftrightarrow \sum \frac{a^2-1}{a^2+11}\geq 0\Leftrightarrow \sum (a-1).\frac{a+1}{a^2+11}\geq 0$

Vì bất đẳng thức hoán vị nên ta giả sử $a\geq b\geq c\geq d$

          $\Rightarrow \frac{a+1}{a^2+11}\geq \frac{b+1}{b^2+11}\geq \frac{c+1}{c^2+11}\geq \frac{d+1}{d^2+11}$

Áp dụng bất đẳng thức Chebyshev ta có:  $\sum (a-1).\frac{a+1}{a^2+11}\geq \frac{1}{4}(\sum a-4).\sum \frac{a+1}{a^2+11}=0$

Vậy có đpcm

Ví dụ $a=1.97749, b=1.12331, c=0.468045$ thì bất đẳng thức này sai.




#545903 Tìm tất cả các cặp số nguyên dương a và b sao cho $\frac{a^2-2...

Đã gửi bởi dogsteven on 24-02-2015 - 17:33 trong Số học

SAO LẠI CÓ CHỖ NÀy $2(a+b)\geq ab+2$

$b|a$ chẵng lẽ $a<b$




#545883 Chứng minh bất đẳng thức

Đã gửi bởi dogsteven on 24-02-2015 - 16:49 trong Bất đẳng thức và cực trị

$\dfrac{a^2+b^2+c^2}{ab+bc+ca} \geqslant \dfrac{2a^2+b^2+c^2}{(a+b)(a+c)}$ và giả sử $a=\text{min}\{a,b,c\}$

Do đó ta cần chứng minh $(2a^2+b^2+c^2)(b+c)+8abc\geqslant 2(a+b)(b+c)(c+a)\Leftrightarrow (b+c-2a)(b-c)^2\geqslant 0$ luôn đúng.




#545852 $\frac{a}{b}+\frac{b}{c...

Đã gửi bởi dogsteven on 24-02-2015 - 13:37 trong Bất đẳng thức và cực trị

$VT\geqslant \dfrac{(a+b+c)^2}{ab+bc+ca}$. Chuẩn hóa $a+b+c=3$ và đặt $x=ab+bc+ca$

BDT này còn quá lỏng.




#545735 $\frac{a}{b}+\frac{b}{c...

Đã gửi bởi dogsteven on 23-02-2015 - 20:09 trong Bất đẳng thức và cực trị

Ở đây $VT\geqslant \dfrac{9(a^2+b^2+c^2)}{(a+b+c)^2}\geqslant \dfrac{3\sqrt{3(a^2+b^2+c^2)}}{a+b+c}$




#545733 $\frac{a}{b}+\frac{b}{c...

Đã gửi bởi dogsteven on 23-02-2015 - 20:08 trong Bất đẳng thức và cực trị

Áp dụng bổ đề $4(a+b+c)^3\geqslant 27(a^2b+b^2c+c^2a+abc)$ hoặc $x^2+y^2+z^2+6\geqslant \dfrac{3}{2}(x+y+z+x^{-1}+y^{-1}+z^{-1})$

Tại đây.

Cách 2 bay link rồi.




#545656 Tìm $x,y,z$ nguyên dương để $P=\frac{1}{2...

Đã gửi bởi dogsteven on 23-02-2015 - 15:31 trong Bất đẳng thức - Cực trị

Mod xóa giúp, đọc nhầm thành GTLN.




#545652 $P=\frac{(a+b+c)^{2}}{5}+\frac...

Đã gửi bởi dogsteven on 23-02-2015 - 15:30 trong Bất đẳng thức và cực trị

 

Ta có:$\frac{1}{{ab}} + \frac{1}{{bc}} + \frac{1}{{ca}} \ge \frac{9}{{ab + bc + ca}} \ge 3$

            $ \Leftrightarrow a + b + c \ge 3abc$

Vì ${a^2} + {b^2} + {c^2} = 3$ nên ta có:
$P - \frac{{93}}{{35}} = \left[ {\frac{{{{\left( {a + b + c} \right)}^2}}}{5} - \frac{9}{5}} \right] + \left( {\frac{{{a^3} + {b^3} + {c^3}}}{{3abc}} - 1} \right) + \left[ {\frac{1}{7} - \frac{{{a^2} + {b^2} + {c^2}}}{{7(ab + bc + ca)}}} \right]$
$ = \left[ {\frac{{{{\left( {a + b + c} \right)}^2}}}{5} - \frac{{3({a^2} + {b^2} + {c^2})}}{5}} \right] + \left( {\frac{{{a^3} + {b^3} + {c^3}}}{{3abc}} - 1} \right) + \left[ {\frac{1}{7} - \frac{{{a^2} + {b^2} + {c^2}}}{{7(ab + bc + ca)}}} \right]$
$\,\,\, = \left[ {{{\left( {a - b} \right)}^2} + {{\left( {b - c} \right)}^2} + {{\left( {c - a} \right)}^2}} \right]\left[ {\frac{{ - 1}}{5} + \frac{{a + b + c}}{{6abc}} - \frac{1}{{14(ab + bc + ca)}}} \right]$
Đặt p = a+b+c, q=ab+bc+ca, r=abc ta có:$p \ge 3r$
Ta đi chứng minh:$\frac{p}{{6r}} - \frac{1}{{14q}} - \frac{1}{5} > 0$

Thật vậy ta có:

$\frac{p}{{6r}} - \frac{1}{{14q}} - \frac{1}{5} > 0$

$ \Leftrightarrow \frac{{7pq - 3r}}{{42qr}} > \frac{1}{5}$

$ \Leftrightarrow 35pq - 15r > 42qr$

$ \Leftrightarrow 35pq > 15r + 42qr$(đúng vì $pq \ge 9r \Rightarrow \frac{5}{3}pq \ge 15r$ và $p \ge 3r \Rightarrow pq \ge 3qr \Rightarrow 14pq \ge 42qr$)

Suy ra $P \ge \frac{{93}}{{35}}$

Dấu “=” xảy ra khi và chỉ khi a=b=c=1.

Vậy GTNN của P là $\frac{93}{35}$ xay ra ki và chỉ khi a=b=c=1

 

 

Đặt $x=ab+bc+ca\in [0,3]$. 

Theo bất đẳng thức Chevbyshev: $\dfrac{a^3+b^3+c^3}{3abc}\geqslant \dfrac{(a+b+c)^2}{(ab+bc+ca)^2}$

Do đó: $P\geqslant \dfrac{3+2x}{5}+\dfrac{3+2x}{x^2}+\dfrac{3}{7x}=\dfrac{(x-3)\left[2(x-3)(7x+1)-29\right]}{35x^2}+\dfrac{309}{105}\geqslant \dfrac{309}{105}$




#545636 CMR:$\frac{1}{x^{k}}+\frac{...

Đã gửi bởi dogsteven on 23-02-2015 - 14:21 trong Bất đẳng thức và cực trị

Chọn $x=x^k ,y=y^k, z=z^k$




#545421 $(\sum ab)^{2}+3\geq 3(\sum ab)+3abc$

Đã gửi bởi dogsteven on 22-02-2015 - 20:46 trong Bất đẳng thức và cực trị

Chọn $a=b=\dfrac{1}{2}$ và $c=2$ thì BDT sai.




#545406 CMR: $\dfrac{a}{b+c-a}+\dfrac{b}...

Đã gửi bởi dogsteven on 22-02-2015 - 20:06 trong Bất đẳng thức và cực trị

Áp dụng bất đẳng thức AM-GM ta được $A\geqslant 3\sqrt[3]{\dfrac{abc}{(b+c-a)(c+a-b)(a+b-c)}}$

Do đó ta cần chứng minh $abc\geqslant (b+c-a)(c+a-b)(a+b-c)$

Hướng 1. Khai triển trực tiếp được bất đẳng thức tương đương là $(a+b-c)^2(a-b)^2+c(a-c)(b-c)\geqslant 0$ luôn đúng nếu $c=\text{min}\{a,b,c\}$

Hướng 2. Áp dụng bất đẳng thức AM-GM $\prod \sqrt{(c+a-b)(a+b-c)} \leqslant \prod \dfrac{c+a-b+a+b-c}{2}=abc$